Let ${P_n}$ be the sequence of all consecutive prime numbers. Is $\sum_{n\geq 1}\frac{1}{p_n}$ convergent?

- 2,909

- 2,269
-
1See http://math.stackexchange.com/questions/94012/sum-of-reciprocal-prime-numbers, http://math.stackexchange.com/q/258337/, http://math.stackexchange.com/q/15946/, http://math.stackexchange.com/q/220386/. At those questions you will find the answer to your question, and much more. – Jonas Meyer May 09 '13 at 03:02
4 Answers
Your wording is ambiguous. Twin primes refer to pairs of primes $p,p+2.$ It is not known whether the collection of twin primes is finite or infinite. One reason it is not known is that the harmonic sum you write converges for the twin primes.

- 139,541
It doesn't. This is a famous result by Euler. An outstanding proof is found for example in Aigner, Ziegler "Proofs from THE BOOK".

- 27,812
The series diverges. You will find multiple proofs here, including an especially compact and elegant proof due to Paul Erdos.

- 2,909
The sum "behaves like" $\sum \frac{1}{n \log n}$ so that the sum of the first $n$ terms is approximately $ \log \log n$. The links to related questions in the first comment can tell more.
edit: I'll leave this up as it is non-duplicative of the linked material, but Will Jagy makes a convincing case that "consecutive" means "twin" and then it is the famous theorem by Brun (heuristically corresponding to convergence of $\int \frac{dn}{n (\log n)^2}$).

- 35,436